Đến nội dung

Hình ảnh

Topic: [LTDH] Mỗi ngày hai bất đẳng thức.


  • Chủ đề bị khóa Chủ đề bị khóa
Chủ đề này có 215 trả lời

#141
tritanngo99

tritanngo99

    Đại úy

  • Điều hành viên THPT
  • 1644 Bài viết

Giúp e 2 bài này đi ,mai em phải nộp rồi! Cho a,b,c>0,abc$\geq$1 CM:$\sum cyc\frac{1}{a^3+2b^3+6}\leq \frac{1}{3}$

 

a,b,c>0.CM: $\sum cyc\frac{1}{(a+b)^2+4abc}+\frac{a^2+b^2+c^2}{8}\geq \sum cyc\frac{1}{a+3}

Do hoangpro1811 là thành viên mới nên chưa đọc rõ nội quy Topic, Hi vọng lần sau em chú ý hơn.

Dưới đây là lời giải hai bài trên:

Bài 1: Romania JBMO TST 2016:http://www.artofprob...1311524p7027737

Bài 2: JBMO 2016:http://www.artofprob...63180p6565536..



#142
tritanngo99

tritanngo99

    Đại úy

  • Điều hành viên THPT
  • 1644 Bài viết

Dưới đây là lời giải bài 61 và bài 62:

Bài 61: Viết lại $P$ như sau: $P=(x+y+1)(\frac{1}{x+1})+\frac{z^3+1}{3(xy+1)}-2$.

Áp dụng bổ đề: Với $x,y>0$ thỏa mãn: $xy\ge 1$ thì ta có: $\frac{1}{x+1}+\frac{1}{y+1}\ge \frac{2}{1+\sqrt{xy}}$.

Áp dụng $AM-GM$: $\left\{\begin{matrix} x+y+1\ge 2\sqrt{xy}+1\\z^3+2=z^3+1+1\ge 3z\ge 3  \end{matrix}\right.$.

$\implies P\ge (2\sqrt{xy}+1).\frac{2}{1+\sqrt{xy}}+\frac{1}{xy+1}-2$.

Đặt $t=\sqrt{xy}\implies t\ge 1$.

Ta có: $P\ge \frac{2(2t+1)}{t+1}+\frac{1}{t^2+1}-2=\frac{2t}{t+1}+\frac{1}{t^2+1}=f(t)$.

$f'(t)=\frac{2}{(t+1)^2}-\frac{2t}{(t^2+1)^2}=\frac{2(t-1)^2+t^2+t+1}{(t+1)^2(t^2+1)^2}\ge 0\forall t\ge 1$.

Do đó: $f(t)\ge f(1)=\frac{3}{2}$.

Suy ra: $P\ge \frac{3}{2}$. Vậy $Min(P)=\frac{3}{2}$.

Có thể tham khảo thêm cách của anh MrS.

Bài 62: Rút: $y=\frac{x+z}{1-xz}(\text{ vì} xz\ne 1)$.

Khi đó: $P=\frac{-2z(z+2x-x^2z)}{(1+x^2)(1+z^2)}-\frac{4z}{\sqrt{z^2+1}}+\frac{3z}{(z^2+1)\sqrt{z^2+1}}$.

Áp dụng $BCS$ ta có: $x^2z-z-2x=z(x^2-1)-2x\le \sqrt{[(x^2-1)^2+4x^2](z^2+1)}=(x^2+1)\sqrt{z^2+1}$.

Vậy, ta có: $P\le \frac{2z}{\sqrt{1+z^2}}-\frac{4z}{\sqrt{z^2+1}}+\frac{3z}{(z^2+1)\sqrt{z^2+1}}$.

Phần còn lại dành cho bạn đọc.



#143
tritanngo99

tritanngo99

    Đại úy

  • Điều hành viên THPT
  • 1644 Bài viết

Tiếp theo: 

Bài 63: Cho $a,b,c$ là các số thực không âm thỏa mãn: $a+b+c=1$. Tìm $GTNN$ của biểu thức:

$P=\sqrt{\frac{a+bc}{1+\sqrt{bc}}}+\sqrt{\frac{b+ca}{1+\sqrt{ca}}}+\sqrt{2c+5}$.

Bài 64: Cho $a,b,c>0$ thỏa mãn: $(a+c)(b+c)=4c^2$. Tìm GTNN của biểu thức:

$A=\frac{4a}{b+c}+\frac{4b}{c+a}-\frac{2ab}{c^2}-\frac{\sqrt{7c^2-3ab}}{c}$



#144
tritanngo99

tritanngo99

    Đại úy

  • Điều hành viên THPT
  • 1644 Bài viết

Dưới đây là lời giải bài 63 và bài 64:

Lời giải bài 63:

Ta có đánh giá như sau: $a+bc\ge a(a+b+c)\ge a^2+2a\sqrt{bc}\ge a^2(1+\sqrt{bc})\implies \sqrt{\frac{a+bc}{1+\sqrt{bc}}}\ge a$.

Tương tự ta có: $\sqrt{\frac{b+ca}{1+\sqrt{ca}}}\ge b$.

Vậy nên: $P\ge a+b+\sqrt{2c+5}=1-c+\sqrt{2c+5}$.

Xét hàm số: $f(c)=\sqrt{2c+5}-c+1;c\in [0;1]$.

Dễ dàng có: $f(c)$ nghịch biến. Vậy $f(c)\ge f(1)=\sqrt{7}$.

Đẳng thức xảy ra khi $a=b=0,c=1$.

Lời giải bài 64: Giả thiết bài toán tương đương với: $(\frac{a}{c}+1)(\frac{b}{c}+1)=4$.

Bây giờ, ta có: $A=\frac{4.\frac{a}{c}}{\frac{b}{c}+1}+\frac{4.\frac{b}{c}}{\frac{a}{c}+1}-2.\frac{a}{c}.\frac{b}{c}-\sqrt{7-3.\frac{a}{c}.\frac{b}{c}}$.

Đặt $(\frac{a}{c};\frac{b}{c})\to (x;y)(x,y>0)\implies (x+1)(y+1)=4$.

Áp dụng $AM-GM$ ta có: $(x+1)(y+1)=4\iff 3=xy+x+y\ge xy+2\sqrt{xy}\implies 0<xy\le 1.$.

Và dễ dàng chứng minh được: $x+y\ge 2$

Khi đó: $A=\frac{4x}{y+1}+\frac{4y}{x+1}-2xy-\sqrt{7-3xy}$

$=\frac{4x(x+1)+4y(y+1)}{(x+1)(y+1)}-2xy-\sqrt{7-3xy}$.

$=x^2+y^2+x+y-2xy-\sqrt{7-3xy}$.

$=(x+y)^2+x+y+xy-5xy-\sqrt{7-3xy}\ge 4+3-5xy-\sqrt{7-3xy}$.

$\implies A\ge 7-5xy-\sqrt{7-3xy}$.

Đặt $t=xy\implies t\in (0;1]$.

Xét hàm: $f(t)=7-5t-\sqrt{7-3t},t\in (0;1]$.

$f'(t)=\frac{3}{2\sqrt{7-3t}}-5<0\forall t\in (0;1]$.

Suy ra $f(t)\ge f(1)=0$.

Vậy nên: $A\ge 0$. Dấu $=$ xảy ra tại $a=b=c$.



#145
tritanngo99

tritanngo99

    Đại úy

  • Điều hành viên THPT
  • 1644 Bài viết

Tiếp theo:

Bài 65: Cho $a,b,c>0$ thỏa mãn: $2a+4b+3c^2=68$. Tìm giá trị nhỏ nhất của biểu thức: $P=a^2+b^2+c^3$.

Bài 66: Cho $a,b,c>0$. Tìm GTNN của biểu thức:

$P=\frac{bc}{(a+c)(a+b)}-\frac{4abc}{(a+b)(b+c)(c+a)}$



#146
leminhnghiatt

leminhnghiatt

    Thượng úy

  • Thành viên
  • 1078 Bài viết

B 65: $$c^3+c^3+64 \geq 3\sqrt[3]{c^6.64}=12c^2$$

$$2(b^2+16) \geq 16b$$

$$2(a^2+4) \geq 8a$$

 

Cộng vế với vế: $$2(a^2+b^2+c^3)+104 \geq 4(2a+4b+3c^2)=272$$

 

$$\rightarrow a^2+b^2+c^3 \geq 84$$

 

Dấu "=" $\iff (a,b,c)=(2;4;4)$


Bài viết đã được chỉnh sửa nội dung bởi leminhnghiatt: 29-09-2016 - 20:48

Don't care


#147
hanguyen445

hanguyen445

    Thượng sĩ

  • Thành viên
  • 240 Bài viết

Tiếp theo:

Bài 65: Cho $a,b,c>0$ thỏa mãn: $2a+4b+3c^2=68$. Tìm giá trị nhỏ nhất của biểu thức: $P=a^2+b^2+c^3$.

Bài 66: Cho $a,b,c>0$. Tìm GTNN của biểu thức:

$P=\frac{bc}{(a+c)(a+b)}-\frac{4abc}{(a+b)(b+c)(c+a)}$

ĐÓNG GÓP BÀI 66:

Tìm điểm rơi:

Bài toán đối xứng 2 biến $b,c$ nên dự đoán điểm rơi $b=c$

+ Với $b=c\Rightarrow P=\dfrac{b^2-2ab}{(a+b)^2}\ge\dfrac{-1}{3}\Rightarrow (a-2b)^2\ge 0$ điều này luôn đúng.

+ Vậy điểm rơi bài toán $a=2b=2c$

Ta đi chứng minh $P\ge\dfrac{-1}{3}$.

$$P\ge -\dfrac{1}{3}\Leftrightarrow 3bc(b+c)-12abc+(a+b)(b+c)(c+a)\ge 0$$

$$\Leftrightarrow 4bc(b+c)+a^2(b+c)+a(b^2+c^2)-10abc\ge 0(@)$$

Ta có:

$$\begin{cases} 4bc(b+c)+a^2(b+c)\ge 4a(b+c)\sqrt{bc}\ge 8abc\\a(b^2+c^2)\ge 2abc\end{cases}$$

Cộng vế hai BĐT trên suy ra:

$VT(@)\ge 10abc-10abc=0$. Do đó (@) được chứng minh.

Vậy $Min P=-\dfrac{1}{3}$ khi $a=2b=2c$


Đề thi chọn đội tuyển  HSG:

http://diendantoanho...date-2016-2017/

Topic thảo luận bài toán thầy Hùng:

http://diendantoanho...topicfilter=all

Blog Thầy Trần Quang Hùng

http://analgeomatica.blogspot.com/

Hình học: Nguyễn Văn Linh

https://nguyenvanlin...ss.com/2016/09/

Toán học tuổi trẻ:

http://www.luyenthit...chi-thtt-online

Mathlink:http://artofproblemsolving.com

BẤT ĐẲNG THỨC:

http://diendantoanho...-đẳng-thức-vmf/

http://diendantoanho...i-toán-quốc-tế/

 


#148
hanguyen445

hanguyen445

    Thượng sĩ

  • Thành viên
  • 240 Bài viết

Tiếp theo:

Bài 65: Cho $a,b,c>0$ thỏa mãn: $2a+4b+3c^2=68$. Tìm giá trị nhỏ nhất của biểu thức: $P=a^2+b^2+c^3$.

Bài 66: Cho $a,b,c>0$. Tìm GTNN của biểu thức:

$P=\frac{bc}{(a+c)(a+b)}-\frac{4abc}{(a+b)(b+c)(c+a)}$

Góp ý cách 2 cho bài 65:

Giả sử điểm rơi bài toán $(a;b;c)=(x;y;z)$

Ta có $a^2\ge 2ax-x^2;b^2\ge 2ay-y^2;c^3\ge\dfrac{3}{2} zc^2-\dfrac{1}{2} z^3$

Suy ra $P\ge 2ax+2by+\dfrac{3}{2}zc^2-(x^2+y^2+\dfrac{z^3}{2})$

Ta chọn hệ số sao cho $\dfrac{2x}{2}=\dfrac{2y}{4}=\dfrac{3z}{6}=k\Rightarrow (x;y;z)=(k;2k;2k)$

Thay $(a;b;c)=(k;2k;2k)$ vào $2a+4b+3c^2=68\Rightarrow k=2$

Suy ra $P\ge 84\Rightarrow Min P=84$


Bài viết đã được chỉnh sửa nội dung bởi hanguyen445: 30-09-2016 - 14:58

Đề thi chọn đội tuyển  HSG:

http://diendantoanho...date-2016-2017/

Topic thảo luận bài toán thầy Hùng:

http://diendantoanho...topicfilter=all

Blog Thầy Trần Quang Hùng

http://analgeomatica.blogspot.com/

Hình học: Nguyễn Văn Linh

https://nguyenvanlin...ss.com/2016/09/

Toán học tuổi trẻ:

http://www.luyenthit...chi-thtt-online

Mathlink:http://artofproblemsolving.com

BẤT ĐẲNG THỨC:

http://diendantoanho...-đẳng-thức-vmf/

http://diendantoanho...i-toán-quốc-tế/

 


#149
tritanngo99

tritanngo99

    Đại úy

  • Điều hành viên THPT
  • 1644 Bài viết

Lời giải khác cho bài 66 như sau:

Đặt: $(x;y)\to (\frac{a}{b};\frac{a}{c})(x,y>0)$.

Ta có: $P=\frac{1}{(x+1)(y+1)}(1-\frac{4xy}{x+y})$.

Theo $AM-GM$, ta có:

$(x+y)(x+1)(y+1)=(x^2y+4y)+(xy^2+4x)+(x^2+y^2)+2xy-3(x+y)\ge 12xy-3(x+y)$.

Suy ra: $1-\frac{4xy}{x+y}\ge -\frac{1}{3}(x+1)(y+1)$.

Vậy $P\ge \frac{-1}{3}, $ dấu $=$ xảy ra khi $x=y=2\iff c=b=\frac{a}{2}$.



#150
tritanngo99

tritanngo99

    Đại úy

  • Điều hành viên THPT
  • 1644 Bài viết

Tiếp theo: 

Bài 67: Cho $a,b,c>0$ thỏa mãn: $a^2+b^2+c^2=3$. Chứng minh rằng: $\sum_{cyc}\frac{1}{1+(ab)^2}\ge \frac{9}{2(a+b+c)}$.

Bài 68: Cho $x,y,z>0$ thỏa mãn: $x^3+8y^3+z(x^2+4y^2)=6xyz$. Tìm GTNN: $P=\frac{x}{2y+z}+\frac{2y}{z+x}+\frac{2z}{x+y}$



#151
baopbc

baopbc

    Himura Kenshin

  • Thành viên nổi bật 2016
  • 410 Bài viết

Bài 67. Theo bất đẳng thức $\text{Cauchy}$,

Ta có $\frac{1}{1+(ab)^2}=\frac{1+(ab)^2}{1+(ab)^2}-\frac{(ab)^2}{1+(ab)^2}=1-\frac{(ab)^2}{1+(ab)^2}\geq 1-\frac{ab}{2}$

Thiết lập các bất đẳng thức tương tự ta cần chứng minh 

\[3-\frac{ab+bc+ca}{2}\geq \frac{9}{2(a+b+c)}\]

Đặt $a+b+c=x$, theo giả thiết thì $a^2+b^2+c^2=3$ nên $ab+bc+ca=\frac{x^2-3}{2}$

Do đó bất đẳng thức cần chứng minh tương đương với

\[3-\frac{x^2-3}{4}\geq \frac{9}{2x}\Leftrightarrow 12x-x^3+3x-18\geq 0\Leftrightarrow x^3-15x+18\leq 0\Leftrightarrow (x-3)(x^2+3x-6)\leq 0\]

Do $a^2+b^2+c^2=3$ nên $\sqrt{3}<x\leq 3$ suy ra $x-3\leq 0,x^2+3x-6>0$. Bất đẳng thức được chứng minh.

Dấu đẳng thức xảy ra khi và chỉ khi $a=b=c=1$.



#152
tritanngo99

tritanngo99

    Đại úy

  • Điều hành viên THPT
  • 1644 Bài viết

Bài 67 baopbc đã cho lời giải đúng. Dưới đây là lời giải bài 68:

Đặt: $(x;2y;z)\to (a;b;c)$. Điều kiện trở thành: $a^3+b^3+c(a^2+b^2)=3abc$.

Lại có: $a^2+b^2\ge 2ab;a^3+b^3\ge ab(a+b)\implies c\ge a+b$.

Bây giờ $P$ viết lại thành: $P=\frac{a}{b+c}+\frac{b}{c+a}+\frac{2c}{a+b}$

$\implies P\ge \frac{(a+b)^2}{2ab+c(a+b)}+\frac{2c}{a+b}\ge \frac{(a+b)^2}{\frac{(a+b)^2}{2}+c(a+b)}+\frac{2a}{a+b}$.

$\implies P\ge \frac{2(a+b)}{a+b+2c}+\frac{2a}{a+b}=2.\frac{1}{1+\frac{2c}{a+b}}+\frac{2c}{a+b}$.

Đặt $t=\frac{2c}{a+b}\ge 2$. Khảo sát ta được: $MinP=\frac{8}{3}$.



#153
tritanngo99

tritanngo99

    Đại úy

  • Điều hành viên THPT
  • 1644 Bài viết

Tiếp theo:

Bài 69: Cho $a,b,c$ là các số thực dương thỏa mãn: $a+2b-c>0,a^2+b^2+c^2=ab+bc+ca+2$. Tìm GTLN của biểu thức:

$P=\frac{a+c+2}{a(b+c)+a+b+1}-\frac{a+b+1}{(a+c)(a+2b-c)}$.

Bài 70: Cho $x,y,z$ là các số thực dương thỏa mãn: $x+y+z=xyz$. Chứng minh rằng:

$(x^2-1)(y^2-1)(z^2-1)\le \sqrt{(x^2+1)(y^2+1)(z^2+1)}$.



#154
Namvip

Namvip

    Hạ sĩ

  • Thành viên
  • 50 Bài viết

Bài 70  Bài giải :

$a=\frac{1}{x};b=\frac{1}{y};c=\frac{1}{z}=>ab+bc+ca=1$

BĐT <=> $(1-a^{2})(1-b^{2})(1-c^{2})\leq abc\sqrt{(1+a^{2})(1+b^{2})(1+c^{2})}$

$(1-a^{2})(1-b^{2})\leq (1-ab)^{2}<=>(a-b)^{2}\geq 0$

CMTT 

=>$\prod (1-a^{2})\leq \prod (1-ab)$

$\prod (1-ab)=\prod (bc+ca)=abc\prod(a+b)$

Lại có 

$\sqrt{(1+a^{2})(1+b^{2})(1+c^{2})}=\sqrt{\prod (a+b)(a+c)}=\prod (a+b)$

$=>(1-a^{2})(1-b^{2})(1-c^{2})\leq abc\sqrt{(1+a^{2})(1+b^{2})(1+c^{2})}$

$=>(x^{2}-1)(y^{2}-1)(z^{2}-1)\leq \sqrt{(x^{2}+1)(y^{2}+1)(z^{2}+1)}$

Vậy bđt đã cho được chứng minh 



#155
hanguyen445

hanguyen445

    Thượng sĩ

  • Thành viên
  • 240 Bài viết

Tiếp theo:

Bài 69: Cho $a,b,c$ là các số thực dương thỏa mãn: $a+2b-c>0,a^2+b^2+c^2=ab+bc+ca+2$. Tìm GTLN của biểu thức:

$P=\frac{a+c+2}{a(b+c)+a+b+1}-\frac{a+b+1}{(a+c)(a+2b-c)}$.

Bài 70: Cho $x,y,z$ là các số thực dương thỏa mãn: $x+y+z=xyz$. Chứng minh rằng:

$(x^2-1)(y^2-1)(z^2-1)\le \sqrt{(x^2+1)(y^2+1)(z^2+1)}$.

Bài 69:

Thứ nhất :

 \[\left( {a + c} \right)\left( {a + 2b - c} \right) \leqslant {\left( {a + b} \right)^2}\]

Thứ hai:

\[{a^2} + {b^2} + {c^2} = ab + bc + ac + 2 \to 1 = \frac{{{a^2} + {b^2} + {c^2} - ab - bc - ac}}{2}\]

\[ \Rightarrow a\left( {b + c} \right) + 1 = a\left( {b + c} \right) + \frac{{{a^2} + {b^2} + {c^2} - ab - bc - ac}}{2}\]

\[ = \frac{{{a^2} + \left( {{b^2} + {c^2}} \right) + ab + ac - bc}}{2} \geqslant \frac{{{a^2} + 2bc + ab + ac - bc}}{2}\]

\[ = \frac{{{a^2} + ab + ac + bc}}{2} = \frac{{\left( {a + b} \right)\left( {a + c} \right)}}{2}\]

\[ \Rightarrow M = a\left( {b + c} \right) + a + b + 1 \geqslant a + b + \frac{{\left( {a + b} \right)\left( {a + c} \right)}}{2} = \frac{{\left( {a + b} \right)\left( {a + c + 2} \right)}}{2}\]

Suy ra:

\[P \leqslant \frac{{2\left( {a + c + 2} \right)}}{{\left( {a + b} \right)\left( {a + c + 2} \right)}} - \frac{{a + b + 1}}{{{{\left( {a + b} \right)}^2}}}\]

\[ = \frac{2}{{a + b}} - \frac{{a + b + 1}}{{{{\left( {a + b} \right)}^2}}} \leqslant  - \frac{1}{4} \Leftrightarrow {\left( {\frac{1}{{a + b}} - \frac{1}{2}} \right)^2} \geqslant 0\]

Luôn đúng.

Vậy Max P=$-\dfrac{1}{4}$

Khi

\[\left( {a;b;c} \right) = \left\{ {\left( {\frac{{2 - \sqrt 2 }}{2};\frac{{2 + \sqrt 2 }}{2};\frac{{2 + \sqrt 2 }}{2}} \right);\left( {\frac{{2 + \sqrt 2 }}{2};\frac{{2 - \sqrt 2 }}{2};\frac{{2 - \sqrt 2 }}{2}} \right)} \right\}\]


Đề thi chọn đội tuyển  HSG:

http://diendantoanho...date-2016-2017/

Topic thảo luận bài toán thầy Hùng:

http://diendantoanho...topicfilter=all

Blog Thầy Trần Quang Hùng

http://analgeomatica.blogspot.com/

Hình học: Nguyễn Văn Linh

https://nguyenvanlin...ss.com/2016/09/

Toán học tuổi trẻ:

http://www.luyenthit...chi-thtt-online

Mathlink:http://artofproblemsolving.com

BẤT ĐẲNG THỨC:

http://diendantoanho...-đẳng-thức-vmf/

http://diendantoanho...i-toán-quốc-tế/

 


#156
tritanngo99

tritanngo99

    Đại úy

  • Điều hành viên THPT
  • 1644 Bài viết

Bài 69 và bài 70: Hai bạn hanguyen45Namvip đã cho lời giải đúng. Ở bài 69. Ta cần chú ý đến bất đẳng thức quan trọng sau:

$(1-a^2)(1-b^2)\le (1-ab)^2\iff (a-b)^2\ge 0$.

Tiếp theo:

Bài 71: Chứng minh rằng với mọi $a,b,c>0$ thỏa mãn: $a+b+c=3$, ta có: $\sqrt[3]{\frac{a^3+b^3+c^3}{3}}+8\sqrt[3]{abc}\le 9$.

Bài 72: Cho các số thực dương $x,y,z$. Tìm GTNN của biểu thức:

$P=\frac{24}{13x+12\sqrt{xy}+16\sqrt{yz}}-\frac{3}{\sqrt{x+y+z}}$



#157
anhquannbk

anhquannbk

    Sĩ quan

  • Thành viên
  • 477 Bài viết

Bài 72:

Áp dụng BĐT $ AM-GM $ ta

: $ 12\sqrt{xy}+ 16\sqrt{yz}=2.\sqrt{3x}.\sqrt{12y} \le 3x +12y$

$ 16\sqrt{yz}=2.2\sqrt{y}.4\sqrt{z} \le 4y +16z $

Suy ra $ 13x+12\sqrt{xy}+16\sqrt{yz} \le 16(x+y+z) $

$ \Longrightarrow P \ge \dfrac{24}{16(x+y+z)}-\dfrac{3}{\sqrt{x+y+z}} =\dfrac{3}{2(x+y+z)}-\dfrac{3}{\sqrt{x+y+z}} $

Đặt $ t=\sqrt{x+y+z} $

Xét hàm $ f(t)=\dfrac{3}{2t^2}-\dfrac{3}{t} $ với $ t \in (0; +\infty ) $

$ f'(t)= \dfrac{-3}{t^3}+\dfrac{3}{t^2} $

$ f'(t)=0 \iff t=1 $

Min $ P=\dfrac{-3}{2} $, xảy ra khi $ x=\dfrac{24}{29} , y= \dfrac{4}{29}, z= \dfrac{1}{29}$


Bài viết đã được chỉnh sửa nội dung bởi anhquannbk: 02-10-2016 - 20:27


#158
Namvip

Namvip

    Hạ sĩ

  • Thành viên
  • 50 Bài viết

Bài 71 :

Ta có 

$\sqrt[3]{\frac{a^{3}+b^{3}+c^{3}}{3}.1.1}\leq \frac{a^{3}+b^{3}+c^{3}}{9}+\frac{2}{3}$

$8\sqrt[3]{abc.1.1}\leq 8(\frac{abc+2}{3})=\frac{8abc}{3}+\frac{16}{3}$

Lại có 

BĐT Phụ sau $a^{3}+b^{3}+c^{3}+24abc\leq (a+b+c)^{3}$

=>$\frac{a^{3}+b^{3}+c^{3}+24abc}{9}\leq \frac{(a+b+c)^{3}}{9}=3$

=>$\sqrt[3]{\frac{a^{3}+b^{3}+c^{3}}{3}}+8\sqrt[3]{abc}\leq 9$


Bài viết đã được chỉnh sửa nội dung bởi Namvip: 02-10-2016 - 20:12


#159
anhquannbk

anhquannbk

    Sĩ quan

  • Thành viên
  • 477 Bài viết

Bài 73:(Dùng pqr với xét hàm hơi cồng kềnh)

Đặt $ a+b+c=p=3, ab+bc+ac=q, abc=r $

Ta $ a^3+b^3+c^3 = p^3-3pq+3r =27-9q+3r $

Suy ra $ \sqrt[3]{\frac{a^3+b^3+c^3}{3}}+8\sqrt[3]{abc} = \sqrt[3]{9-3q+r} +8 \sqrt[3]{r} $ (*)

$ pq \ge 9r $ nên $ r \le \dfrac{q}{3} $ dẫn đến (*) $ \le \sqrt[3]{9-3q+\dfrac{q}{3}} + 8\sqrt[3]{\dfrac{q}{3}} $

Xét hàm $ f(q)= \sqrt[3]{9-\dfrac{8q}{3}} + 8\sqrt[3]{\dfrac{q}{3}} $ với $ q \in (0; 3] $

$ f'(q) = \dfrac{\dfrac{-8}{3}}{3\sqrt[3]{(9-\dfrac{8q}{3})^2}} + \dfrac{\dfrac{8}{3}}{3\sqrt[3]{(\dfrac{q}{3})^2}}$

$ f'(q)=0 \iff q=3 $

Do đó $ f(q) \le f(3) = 9 $

Vậy $ \sqrt[3]{\frac{a^3+b^3+c^3}{3}}+8\sqrt[3]{abc} \le 9 $, xảy ra khi $ a=b=c=1 $


Bài viết đã được chỉnh sửa nội dung bởi anhquannbk: 02-10-2016 - 20:31


#160
hanguyen445

hanguyen445

    Thượng sĩ

  • Thành viên
  • 240 Bài viết

Bài 69 và bài 70: Hai bạn hanguyen45Namvip đã cho lời giải đúng. Ở bài 69. Ta cần chú ý đến bất đẳng thức quan trọng sau:

$(1-a^2)(1-b^2)\le (1-ab)^2\iff (a-b)^2\ge 0$.

Tiếp theo:

Bài 71: Chứng minh rằng với mọi $a,b,c>0$ thỏa mãn: $a+b+c=3$, ta có: $\sqrt[3]{\frac{a^3+b^3+c^3}{3}}+8\sqrt[3]{abc}\le 9$.

Bài 72: Cho các số thực dương $x,y,z$. Tìm GTNN của biểu thức:

$P=\frac{24}{13x+12\sqrt{xy}+16\sqrt{yz}}-\frac{3}{\sqrt{x+y+z}}$

1 cách bài 71[Không được đẹp mắt]

BĐT tương đương với:

  \[ \Leftrightarrow \sqrt[3]{{\frac{{27 - 9\left( {ab + bc + ac} \right) + 3abc}}{3}}} + 8\sqrt[3]{{abc}} \leqslant 9\]

Ta có:

\[{\left( {ab + bc + ac} \right)^2} \geqslant 3abc\left( {a + b + c} \right) = 9abc \Leftrightarrow abc \leqslant \frac{{{{\left( {ab + bc + ac} \right)}^2}}}{9}\]

\[ \Rightarrow VT \leqslant \sqrt[3]{{9 - 3\left( {ab + bc + ac} \right) + \frac{{{{\left( {ab + bc + ac} \right)}^2}}}{9}}} + 8\sqrt[3]{{\frac{{{{\left( {ab + bc + ac} \right)}^2}}}{9}}}\]

Đặt:

\[x = \sqrt[3]{{\frac{{ab + bc + ac}}{3}}} \leqslant 1 \Leftrightarrow ab + bc + ac = 3{x^3}\]

Suy ra:

\[VT \leqslant \sqrt[3]{{9 - 9{x^3} + {x^6}}} + 8{x^2} \leqslant 9 \Leftrightarrow 9 - 9{x^3} + {x^6} \leqslant 729 - 512{x^6} - 1944{x^2} + 1728{x^4}\]

\[ \Leftrightarrow 513{x^6} - 1728{x^4} - 9{x^3} + 1944{x^2} - 720 \leqslant 0\]

\[ \Leftrightarrow \left( {x - 1} \right)\left( {513{x^5} + 513{x^4} - 1215{x^3} - 1224{x^2} + 720x + 720} \right) \leqslant 0 \Leftrightarrow \left( {x - 1} \right)g\left( x \right) \leqslant 0\left( * \right)\]

Lại có:

\[{\text{g}}\left( x \right) = 513{x^5} + 513{x^4} - 1215{x^3} - 1224{x^2} + 720x + 720\]

\[ = x\left( {513{x^4} - 1215{x^2} + 720} \right) + 513\left( {{x^4} + 1} \right) + 207 - 1224{x^2} > 1026{x^2} - 1224{x^2} + 207\]

\[ > 207 - 198{x^2} > 0\]

luôn đúng do $x\le 1$. Vậy BĐT (*) được chứng minh


Đề thi chọn đội tuyển  HSG:

http://diendantoanho...date-2016-2017/

Topic thảo luận bài toán thầy Hùng:

http://diendantoanho...topicfilter=all

Blog Thầy Trần Quang Hùng

http://analgeomatica.blogspot.com/

Hình học: Nguyễn Văn Linh

https://nguyenvanlin...ss.com/2016/09/

Toán học tuổi trẻ:

http://www.luyenthit...chi-thtt-online

Mathlink:http://artofproblemsolving.com

BẤT ĐẲNG THỨC:

http://diendantoanho...-đẳng-thức-vmf/

http://diendantoanho...i-toán-quốc-tế/

 





1 người đang xem chủ đề

0 thành viên, 1 khách, 0 thành viên ẩn danh